Dummit Foote Chapter 14 Exercises
Dummit Foote Chapter 14 Exercises
Contents
1
1 Chapter 14.1
Question 1.1
1. Show that if the field 𝐾 is generated over 𝐹 by the elements 𝑎1 , ⋯ , 𝑎𝑛 then an automorphism 𝜎 of
𝐾 fixing 𝐹 is uniquely determined by 𝜎(𝑎1 ), ⋯ , 𝜎(𝑎𝑛 ). In particular, show that an automorphism
fixes 𝐾 if and only if it fixes a set of generators for 𝐾
2. Let 𝐺 ≤ Gal(𝐾 /𝐹 ) be a subgroup of the Galois group of the extension 𝐾 /𝐹 and suppose 𝜎1 , ⋯ , 𝜎𝑘
are generators for 𝐺. Show that the subfield 𝐸/𝐹 is fixed by 𝐺 if and only if it is fixed by the
generators 𝜎1 , ⋯ , 𝜎𝑘
Solution.
𝑗 𝑗
(a) Let 𝑥 ∈ 𝐾 = 𝐹 (𝛼1 , ⋯ , 𝛼𝑛 ), then we have that 𝑥 = 𝑎1 𝜃1 + ⋯ + 𝑎𝑚 𝜃𝑚 where 𝜃𝑖 = 𝛼1𝑖,1 ⋯ 𝛼𝑛𝑖,𝑛 . (This is basically
saying that each element in 𝐾 is expressed √ √as a linear combination
√ √ of
√ all possible products of the 𝛼𝑖 ’s,
which is obviously true, for example, ℚ( 2, 3) = {𝑎 + 𝑏 2 + 𝑐 3 + 𝑑 6 ∣ 𝑎, 𝑏, 𝑐, 𝑑 ∈ ℚ})
Then, we have that
𝜎(𝑥) = 𝜎(𝑎1 𝜃1 + ⋯ + 𝑎𝑚 𝜃𝑚 ) = 𝑎1 𝜎(𝜃1 ) + ⋯ + 𝑎𝑚 𝜎(𝜃𝑚 )
since 𝜎 is a homomorphism that fixes 𝐹 . Furthermore,
𝑗 𝑗
𝜎(𝜃𝑖 ) = 𝜎(𝛼1𝑖,1 ⋯ 𝛼𝑛𝑗𝑖,𝑛 ) = 𝜎(𝛼1𝑖,1 ) ⋯ 𝜎(𝛼𝑛𝑗𝑖,𝑛 )
where each 𝛾𝑗 ∈ {𝜎𝑖 ∣ 1 ≤ 𝑖 ≤ 𝑘} (note that 𝛾𝑗 are not necessarily distinct, in fact there are likely to be
repeats), and 𝑛𝑗 ∈ ℤ.
By assumption, each 𝜎𝑖 ↾𝐸 = 1, that is 𝜎𝑖𝑛𝑖 (𝑥) = 𝑥 for any 𝑥 ∈ 𝐸 and 𝑛𝑖 ∈ ℤ. It follows immediately that
𝑚
𝑛
𝜎(𝑥) = (∏ 𝛾𝑗 𝑗 )(𝑥) = 𝑥.
𝑗=1
Question 1.2
Let 𝜏 be the map 𝜏 ∶ ℂ → ℂ defined by 𝜏(𝑎 + 𝑏𝑖) = 𝑎 − 𝑏𝑖. Prove that 𝜏 is an automorphism of ℂ
Solution. It is easily shown that 𝜏 is a homomorphism and that it is bijective and hence 𝜏 is an isomorphism □
Question 1.3
Solution. The fixed field of complex conjugation is 𝐹 = {𝑎 + 𝑏𝑖 ∈ ℂ ∣ 𝜏(𝑎 + 𝑏𝑖) = 𝑎 + 𝑏𝑖}, therefore we need
𝜏(𝑎 + 𝑏𝑖) = 𝑎 − 𝑏𝑖 = 𝑎 + 𝑏𝑖 ⟹ 2𝑏𝑖 = 0 ⟹ 𝑏 = 0 therefore 𝑎 + 𝑏𝑖 ∈ 𝐹 ⟺ 𝑏 = 0. In this case we have
𝑎 + 𝑏𝑖 = 𝑎 ∈ ℝ and therefore 𝐹 = ℝ □
2
Question 1.4
√ √
Prove that ℚ( 2) and ℚ( 3) are not isomorphic
Question 1.5
√ √
Determine the automorphisms of the extension ℚ( 4 2)/ℚ( 2) explicitly
√4 √ √ √
√4 First we note that [ℚ( 2) ∶ ℚ( 2)] = 2 and we have minimal polynomial 𝑥 − 2 ∈ ℚ( 2) with
Solution. 2
Question 1.6
Let 𝑘 be a field
(a) Show that the mapping 𝜑 ∶ 𝑘[𝑡] → 𝑘[𝑡] defined by 𝜑(𝑓 (𝑡)) = 𝑓 (𝑎𝑡 + 𝑏) for fixed 𝑎, 𝑏 ∈ 𝑘, 𝑎 ≠ 0 is an
automorphism of 𝑘[𝑡] which is the identity on 𝑘
(b) Conversely, let 𝜑 be an automorphism of 𝑘[𝑡] which is the identity on 𝑘. Prove that there exist
𝑎, 𝑏 ∈ 𝑘 with 𝑎 ≠ 0 such 𝜑(𝑓 (𝑡)) = 𝑓 (𝑎𝑡 + 𝑏) as in (𝑎)
Solution.
(a) Let 𝑓 (𝑡), 𝑔(𝑡) ∈ 𝑘[𝑡], then we show that 𝜑 is an isomorphism.
𝜑((𝑓 + 𝑔)(𝑡)) = (𝑓 + 𝑔)(𝑎𝑡 + 𝑏) = 𝑓 (𝑎𝑡 + 𝑏) + 𝑔(𝑎𝑡 + 𝑏) = 𝜑(𝑓 (𝑡)) + 𝜑(𝑔(𝑡))
and
𝜑((𝑓 𝑔)(𝑡)) = (𝑓 𝑔)(𝑎𝑡 + 𝑏) = 𝑓 (𝑎𝑡 + 𝑏)𝑔(𝑎𝑡 + 𝑏) = 𝜑(𝑓 (𝑡))𝜑(𝑔(𝑡))
Therefore 𝜑 is a homomorphism. Now, suppose 𝜑(𝑓 (𝑡)) = 𝜑(𝑔(𝑡)). Then 𝑓 (𝑎𝑡 + 𝑏) = 𝑔(𝑎𝑡 + 𝑏) and because
𝑘[𝑎𝑡 + 𝑏] = 𝑘[𝑡] we have that 𝑓 (𝑡) = 𝑔(𝑡). Lastly let 𝑔(𝑡) ∈ 𝑘[𝑡] then take 𝑓 (𝑡) = 𝑔( 𝑎𝑡 − 𝑎𝑏 ) ∈ 𝑘[𝑡] and we
have 𝜑(𝑓 (𝑡)) = 𝜑(𝑓 (𝑎𝑡 + 𝑏)) = 𝑔(𝑎( 𝑎𝑡 − 𝑎𝑏 ) + 𝑏) = 𝑔(𝑡) and therefore 𝜑 is bijective, finally we conclude 𝜑 is
an isomorphism.
Lastly, if 𝑓 (𝑡) = 𝑐 ∈ 𝑘 ⊂ 𝑘[𝑡] then 𝜑(𝑓 (𝑡)) = 𝑓 (𝑎𝑡 + 𝑏) = 𝑐 and therefore 𝜑 is the identity on 𝑘
(b) Suppose 𝜑(𝑓 (𝑡)) = ℎ(𝑡)𝑓 (𝑡) + 𝑔(𝑡) where 𝑔(𝑡), ℎ(𝑡) ∈ 𝑘[𝑡] then because 𝜑 is identity on 𝑘 we would have
𝜑(𝑐) = ℎ(𝑡)𝑐 + 𝑔(𝑡) = 𝑐 ⟹ 𝑔(𝑡) = 0, ℎ(𝑡) = 1 therefore we must have that 𝜑(𝑓 (𝑡)) = 𝑓 (𝑔(𝑡)) for some
𝑔(𝑡) ∈ 𝑘[𝑡].
We want 𝑔(𝑡) = 𝑎𝑡 + 𝑏 therefore we must show that if deg(𝑔(𝑡)) ≥ 2 there is a contradiction.
Suppose deg(𝑔(𝑡)) ≥ 2 this implies that the deg(𝑓 (𝑔(𝑡)) ≥ 2 and therefore this map is not surjective,
therefore we conclude deg(𝑔(𝑡)) ≤ 1.
If deg(𝑔(𝑡)) = 0 then 𝑔(𝑡) = 𝑏 ∈ 𝑘 and this map is not injective.
Finally, we conclude that deg(𝑔(𝑡)) = 1 and therefore 𝑔(𝑡) = 𝑎𝑡 + 𝑏 where 𝑎, 𝑏 ∈ 𝑘 and 𝜑(𝑓 (𝑡)) = 𝑓 (𝑔(𝑡)) =
𝑓 (𝑎𝑡 + 𝑏)
3
□
Question 1.7
(b) Prove that any − 𝑚1 < 𝑎 − 𝑏 < 𝑚1 implies − 𝑚1 < 𝜎(𝑎) − 𝜎(𝑏) < 1
𝑚 for every positive integer 𝑚.
Conclude that 𝜎 is a continuous map on ℝ
(c) Prove that any continuous map on ℝ which is the identity on ℚ is the identity map, hence
Aut(ℝ/ℚ) = 1
Solution.
(a) Let 𝑎 ∈ ℝ be a square. That is, ∃𝑏 ∈ ℝ s.t. 𝑏2 = 𝑎. Then 𝜎(𝑎) = 𝜎(𝑏2 ) = (𝜎(𝑏))2 . That is, 𝜎 takes squares to
squares. Since the only squares in ℝ are the non-negative reals, but 𝜎(𝑎) = 0 ⟹ 𝑎 = 0, so it must be
that 𝜎 takes positive reals to positive reals.
Suppose now that 𝑏 − 𝑎 > 0, then 𝜎(𝑏 − 𝑎) > 0, giving that 𝜎(𝑏) − 𝜎(𝑎) > 0.
(b) Since ∀𝜎 ∈ Aut(ℝ/ℚ), 𝜎 fixes ℚ, then
1 1
− <𝑎−𝑏<
𝑚 𝑚
1 1
𝜎 − < 𝜎(𝑎 − 𝑏) < 𝜎 , 𝜎 preserves order by part (a)
( 𝑚) (𝑚)
2 Chapter 14.2
Question 2.1
√ √
Determine the minimal polynomial over ℚ for the element 2+ 5
√ √ √ √
Solution.
√ ℚ( √ 2 + 5) ⊂ ℚ( 2, 5) which is Galois over ℚ and therefore the roots
√ of√the minimal
√ polynomial
√
are
√ ± 2
√ ± 5 which
√ are
√ all distinct. Hence the minimal polynomial is (𝑥 − ( 2 + 5)(𝑥 + ( 2 + 5))(𝑥 −
( 2 − 5))(𝑥 + ( 2 − 5)) = 𝑥 4 − 14𝑥 2 + 9 □
4
Question 2.2
√ √
Determine the minimal polynomial over ℚ for the element 1 + 3 2 + 3 4
√3 √3 √ √ 2𝜋𝑖
Solution. We have shown in √3 chapter 13 that ℚ(1 √3 +√3 2 +
√3 4) ⊂ ℚ( 3 2) ⊂ ℚ( 3 2, 𝜁 ) where 𝜁 = 𝑒 3 which√is a
Galois extension, therefore 2 must be sent to 2, 2𝜁 , 2𝜁 2 and notice that we only care about where 3 2 is
√ 2 √ √ 3
sent as 3 2 = 3 4, 3 2 = 1.
Knowing this we know that the 3 roots of our minimal polynomial are
√ √
𝑟1 = 1 + 3 2 + 3 4
√ √
𝑟2 = 1 + 3 2𝜁 + 3 4𝜁 2
√ √
𝑟3 = 1 + 3 2𝜁 2 + 3 4𝜁
√ √
Painfully
√3 √expanding (𝑥 −𝑟1√)(𝑥 −𝑟√2 )(𝑥 −𝑟3 ) gives you 𝑥 3 −3𝑥 2 −3𝑥 −1. Alternatively (𝑟1 −1)3 = ( 3 2+ 3 4)3 =
2 + 3 16 + 3 3 32 + 4 = 6 + 6( 3 2 + 3 4) = 6 + 6(𝑟1 − 1) = 6𝑟1 □
Question 2.3
Determine the Galois group of 𝑓 = (𝑥 2 − 2)(𝑥 2 − 3)(𝑥 2 − 5). Determine all subfields of the splitting
field of 𝑓
√ √ √ √ √
Solution. The splitting field of 𝑓 is clearly 𝐾 = ℚ( 2, 3, 5) and any automorphism of 𝐾 will map 𝑎 → ± 𝑎
where 𝑎 ∈ {2, 3, 5} and therefore there are 8 total automorphisms. Now we must show that there are no more
than 8, this is done by noting that | Aut(𝐾 /ℚ)| ≤ [𝐾 ∶ ℚ] = 8, furthermore we can conclude that this
extensions is Galois. The subfields are
√
ℚ( 𝑎) where 𝑎 ∈ {2, 3, 5, 6, 10, 15, 30}
√ √ √ √ √ √ √ √ √ √ √ √ √ √
ℚ( 2, 3), ℚ( 2, 5), ℚ( 3, 5), ℚ( 2, 15), ℚ( 3, 10), ℚ( 5, 6), ℚ( 10, 15)
□
Question 2.4
√
Solution. The splitting field of 𝑥 𝑝 − 2 is 𝐾 = ℚ( 𝑝 2, 𝜁 ) where 𝜁 is the p-th root of unity.
√ √
1. Consider 𝐺1 = Gal(𝐾 /ℚ(𝜁 )) and 𝜏( 𝑝 2) = 𝑝 2𝜁 and it fixes 𝜁 . The order of 𝜏 is 𝑝 and therefore 𝐺1 ≅
⟨𝜏 ⟩ ≅ 𝐶𝑝
√ √
2. Consider 𝐺2 = Gal(𝐾 /ℚ( 𝑝 2)) and 𝜎(𝜁 ) = 𝜁 𝑎 and it fixes 𝑝 2. The order of 𝜎 is 𝑝 − 1 because 𝑎𝑝−1 ≡ 1
(mod 𝑝) and therefore 𝐺2 ≅ ⟨𝜎 ⟩ ≅ 𝐶𝑝−1
Furthermore, we know the following:
√ √
1. [𝐾 ∶ ℚ] = [𝐾 ∶ ℚ( 𝑝 2)][ℚ( 𝑝 2) ∶ ℚ] = (𝑝 − 1)𝑝 is a galois extension and hence |𝐺| = | Gal(𝐾 /ℚ)| =
𝑝(𝑝 − 1)
2. |⟨𝜏 ⟩||⟨𝜎 ⟩| = 𝑝(𝑝 − 1)
3. |⟨𝜏 ⟩ ⋂ ⟨𝜎 ⟩| = 1
|⟨𝜏 ⟩||⟨𝜎 ⟩|
Therefore, using point 2 and 3 and the following |⟨𝜏 ⟩⟨𝜎 ⟩| = we have that 𝐺 = ⟨𝜏 ⟩⟨𝜎 ⟩. Futher-
|⟨𝜏 ⟩ ⋂ ⟨𝜎 ⟩|
more we can notice that 𝐾 𝐺1 /ℚ = ℚ(𝜁 )/ℚ is a galois extension because [ℚ(𝜁 ) ∶ ℚ] = 𝑝 − 1 = | Aut(ℚ(𝜁 )/ℚ)|
and therefore 𝐺1 ⊲ 𝐺 and therefore we have 𝐺 ≅ 𝐶𝑝 ⋊ 𝐶𝑝−1 □
5
Question 2.5
Prove that the Galois group of 𝑥 𝑝 − 2 for 𝑝 a prime is isomorphic to the group of matrices
𝑎 𝑏
(0 1)
where 𝑎, 𝑏 ∈ 𝔽𝑝 , 𝑎 ≠ 0
√ √
Solution. Let 𝐺 = Gal(ℚ( 𝑝 2, 𝜁 )/ℚ). Now notice
√ that√any element 𝜑 ∈ 𝐺 is determined by 𝜑(𝜁 ) and 𝜑( 𝑝 2),
where 𝜑(𝜁 ) = 𝜁 𝑎 for some 1 ≤ 𝑖 ≤ 𝑝 − 1 and 𝜑( 𝑝 2) = 𝑝 2𝜁 𝑏 for some 0 ≤ 𝑏 ≤ 𝑝 − 1 then we define the map
𝑎 𝑏
𝛼∶𝐺→{ where 𝑎, 𝑏 ∈ 𝔽𝑝 , 𝑎 ≠ 0}
(0 1)
𝑎 𝑏
𝛼(𝜑) =
(0 1)
This is a homomorphism and bijective, hence an isomorphism □
Question 2.6
√ √ √
Let 𝐾 = ℚ( 8 2, 𝑖) and let 𝐹1 = ℚ(𝑖), 𝐹2 = ℚ( 2), 𝐹3 = ℚ(− 2). Prove that Gal(𝐾 /𝐹1 ) ≅ ℤ8 , Gal(𝐾 /𝐹2 ) ≅
𝐷8 , Gal(𝐾 /𝐹3 ) ≅ 𝑄8
Solution. We follow the discussion from Chapter 14.2 where we found that
⎧√8 √ ⎧√8 √
⎪
⎪ 2 → 𝜁 82 ⎪
⎪ 2 → 82
⎪ ⎪
Gal(𝐾 /ℚ) = ⟨𝜎, 𝜏 ∶ 𝜎 8 = 𝜏 2 = 1, 𝜎𝜏 = 𝜏𝜎 3 ⟩ where 𝜎 = ⎨𝑖 → 𝑖 and 𝜏 = ⎨𝑖 → −𝑖
⎪
⎪ 5
⎪
⎪ 7
⎩𝜁 → 𝜁
⎪ ⎩𝜁 → 𝜁
⎪
Determine all the subfields of the splitting field of 𝑥 8 − 2 which are Galois over ℚ
Suppose 𝐾 is a Galois extension of 𝐹 of degree 𝑝𝑛 for some prime 𝑝 and some 𝑛 ≥ 1. Show there are
Galois extensions of 𝐹 contained in 𝐾 of degrees 𝑝 and 𝑝𝑛−1
6
Question 2.9
Give an example of fields 𝐹1 , 𝐹2 , 𝐹3 with ℚ ⊂ 𝐹1 ⊂ 𝐹2 ⊂ 𝐹3 , [𝐹3 ∶ ℚ] = 8 and each field if Galois over all
of its subfields with the exception that 𝐹2 is not Galois over ℚ
√ √ √
Solution. Take 𝐹3 = ℚ( 4 2, 𝑖), 𝐹2 = ℚ( 4 2), 𝐹1 = ℚ( 2). Then we have that 𝐹3 is Galois over 𝐹2 , 𝐹1 , ℚ, 𝐹2 is
Galois over 𝐹1 but not Galois over ℚ and 𝐹1 is Galois over ℚ □
Question 2.10
√ √ √
Solution. The splitting field of the polynomial is 𝐾 = ℚ( 8 3, 𝜁 ) = ℚ( 8 3, 2, 𝑖) where
√8 𝜁 √is an 8-th
√8 root
√ of
unity.
√ This√extension is of degree 32 because of the√ following,
√ [𝐾 ∶ ℚ] = [𝐾 ∶ ℚ( 3, 2)][ℚ( 3, 2) ∶
ℚ( 2)][ℚ( 2) ∶ ℚ] = 32 because we can show that 8 3 ∉ ℚ( 2).
Any automorphism of Aut(𝐾 /ℚ) is of the form
√8 √ √ √
3 ↦ 8 3𝜁 𝑖 , 1 ≤ 𝑖 ≤ 7, 2 ↦ ± 2, 𝑖 ↦ ±𝑖
2. 𝜏𝑖 ∶ 𝜁 ↦ 𝜁 𝑖 , for 𝑖 ∈ {3, 5, 7}
and work out the relations. Namely, all automorphisms can be written in the form 𝜎 𝑎 , 𝜎 𝑎 𝜏3 , 𝜎 𝑎 𝜏5 , 𝜎 𝑎 𝜏7 for
0 ≤ 𝑎 ≤ 7, giving exactly 32 automorphisms as desired. □
Suppose 𝑓 (𝑥) ∈ ℤ[𝑥] is an irreducible quartic whose splitting field has Galois group 𝑆4 over ℚ (there
are many such quartics, cf. Section 6). Let 𝜃 be a root of 𝑓 (𝑥) and set 𝐾 = ℚ(𝜃). Prove that 𝐾 is an
extension of ℚ of degree 4 which has no proper subfields. Are there any Galois extensions of ℚ of
degree 4 with no proper subfields?
Question 2.12
√ √
√ √ Note: From Question 2.1
Solution. √ we√ can already see that the splitting field of the polynomial is ℚ( 2+ 5) =
ℚ( 2, 5) and therefore Gal(ℚ( 2 + 5)/ℚ) ≅ 𝐾4 , now we can just confirm the answer.
Solving for 𝑥 2 using the quadratic formula we see that
√
2 14 ± 142 − 4(1)(9) √ √ √
𝑥 = = 7 ± 2 10 = ( 2 ± 5)2
2
√ √
Then,√we have
√ that the√ roots
√ of the polynomial are ± 2± 5 and therefore the splitting field of the polynomial
is ℚ( 2 + 5) = ℚ( 2, 5) which has 4 automorphisms.
Finally, we conclude
{√ √ {√ √
√ √ 2→ 2 2→− 2
Gal(ℚ( 2 + 5)/ℚ) = {1, 𝜎, 𝜏, 𝜎𝜏 = 𝜏𝜎} ≅ 𝐾4 where 𝜎 = √ √ and 𝜏 = √ √
5→− 5 5→ 5
7
Question 2.13
Prove that if the Galois group of the splitting field of a cubic over ℚ is the cyclic group of order 3 then
all the roots of the cubic are real.
Solution. Suppose the 3 roots are not all real, then we must have one real root 𝑟1 and 2 complex roots 𝑧, 𝑧 in
which case the splitting field would be ℚ(𝑟1 , 𝑧) and we have an automorphism of Gal(ℚ(𝑟1 , 𝑧)/ℚ) which would
fix 𝑟1 and send 𝑧 ↦ 𝑧 and therefore 2 would divide | Gal(ℚ(𝑟1 , 𝑧)/ℚ)| and hence Gal(ℚ(𝑟1 , 𝑧)/ℚ) ≇ ℤ3 □
Question 2.14
√ √
Show that ℚ( 2 + 2) is a cyclic quartic field, i.e, is a Galois extension of degree 4 with cyclic Galois
group
√ √ √ √
Solution.
√ Let 𝐾 = ℚ( √ 2 + 2). We find a polynomial with root 𝑥 = 2 + 2 using the following 𝑥 2 =
2 + 2 ⟹ 𝑥 2 − 2 = 2 ⟹ 𝑥 4 − 4𝑥 2 + 4 = 2 ⟹ 𝑥 4 − 4𝑥 2 + 2 which √ is a degree 4 polynomial and is
√
irreducible
√ by Eisenstein criterion, therefore
√ it is the
√
minimum polynomial of 2 + 2 over ℚ. The 4 roots are
√ √
± 2 ± 2 and we can notice that 2 − 2 = √ 2√ ∈ 𝐾 , so all our roots are contained in 𝐾 which makes 𝐾
2+ 2
the splitting field of a separable polynomial (as the roots are distinct) and therefore
√ a Galois
√ Extension of ℚ,
√ √
hence | Aut(𝐾 /ℚ)| = [𝐾 ∶ ℚ] = 4. Furthermore, if 𝜎 ∈ Gal(𝐾 /ℚ) such that 𝜎( 2 + 2) = 2 − 2 we have
that
√ √ √ √ √
√ √ √ √ 2 𝜎( 2) 𝜎(( 2 + 2)2 − 2) − 2 √ √
2
𝜎 ( 2 + 2) = 𝜎( 2 − 2) = 𝜎( √ √ )= √ √ = √ √ =√ √ =− 2− 2
2+ 2 𝜎( 2 + 2) 2− 2 2+ 2
Therefore ord(𝜎) > 2 and it must divide 4, which implies that ord(𝜎) = 4 and therefore Gal(𝐾 /ℚ) ≅ ℤ4 □
Question 2.15
Solution.
√ √
(a) If 𝐷1 , 𝐷2 are not square in 𝐹 this implies that [𝐹 ( 𝐷1 ) ∶ 𝐹 ] = [𝐹 ( 𝐷2 ) ∶ 𝐹 ] = 2 and therefore
√ √ √ √
[𝐾 ∶ 𝐹 ] = [𝐾 ∶ 𝐹 ( 𝐷1 )][𝐹 ( 𝐷1 ) ∶ 𝐹 ] ≤ [𝐹 ( 𝐷1 ) ∶ 𝐹 ][𝐹 ( 𝐷2 ) ∶ 𝐹 ] = 4
√ √ √ √
We then √ have that√[𝐾 ∶ 𝐹 ( 𝐷1 )] ≤ √ 2. To show√that [𝐾 ∶ 𝐹 ( 𝐷1 )] = 2 we show that 𝐷2√∉ 𝐹 ( 𝐷1 ).
Suppose 𝐷2 ∈ 𝐹 ( 𝐷1 ) then we have 𝐷2 = 𝑎+𝑏 𝐷1 where 𝑎, 𝑏 ∈ 𝐹 , this implies 𝐷2 = 𝑎2 +2𝑎𝑏 𝐷1 +𝑏2 𝐷1 ,
because 𝐷2 is not square in 𝐹 we must have 𝑎 = 0 or 𝑏 = 0. If 𝑏 = 0 then 𝐷2 = 𝑎2 which means 𝐷2 is a
square, a contradiction. If 𝑎 = 0 then√ 𝐷2 = √ 𝑏2 𝐷1 ⟹ 𝐷1 𝐷2 = 𝑏2 𝐷12 which means 𝐷1 𝐷2 is a square, a
contradiction. Hence we conclude 𝐷2 ∉ 𝐹 ( 𝐷1 ) and therefore [𝐾 ∶ 𝐹 ] = 4. Furthermore, it is easy to
see that we have 4 automorphisms of 𝐾 fixing 𝐹
{√ √ {√ √ {√ √
𝐷1 ↦ − 𝐷1 𝐷1 ↦ 𝐷1 𝐷1 ↦ − 𝐷1
𝐼𝑑 𝜎 = √ √ 𝜏= √ √ 𝜎𝜏 = 𝜏𝜎 = √ √
𝐷2 ↦ 𝐷2 𝐷2 ↦ − 𝐷2 𝐷2 ↦ − 𝐷2
8
(b) Given that Gal(𝐾 /𝐹 ) ≅ 𝐾4 and 𝐾4 has 3 non-trivial subgroups or order 2; ⟨1, 𝜎⟩, ⟨1, 𝜏⟩, ⟨1, 𝜎𝜏⟩ there will be
correspondingly
√ √3 subfields 𝐸1 , 𝐸2 , 𝐸3 of 𝐾 containing 𝐹 where they are degree 2 extensions of 𝐹 . Let 𝐸1 =
𝐹 ( 𝐷1 ), 𝐸2 = 𝐹 ( 𝐷2 ) where 𝐷1 , 𝐷2 are not square √ in 𝐹 as needed, then the fact that 𝐸1 ≠ 𝐸2 ⟹ 𝐷1 𝐷2 is
not square in 𝐹 from part (a), therefore 𝐸3 = 𝐹 ( 𝐷1 𝐷2 ) is a degree 2 extension √ of 𝐹√. Finally, we have that
𝐸1 𝐸2 is a degree 4 extension over 𝐹 and 𝐸1 , 𝐸2 , 𝐸3 ⊂ 𝐸1 𝐸2 ⟹ 𝐾 = 𝐸1 𝐸2 = 𝐹 ( 𝐷1 , 𝐷2 )
□
Question 2.16
Solution.
9
Question 2.17
Let 𝐾 /𝐹 be any finite extension and let 𝛼 ∈ 𝐾 . Let 𝐿 be a Galois extension of 𝐹 containing 𝐾 and let
𝐻 ≤ Gal(𝐿/𝐹 ) be the subgroup corresponding to 𝐾 . Define the norm of 𝛼 from 𝐾 to 𝐹 to be
𝑁𝐾 /𝐹 (𝛼) = ∏ 𝜎(𝛼)
𝜎
where the product is taken over all the embeddings of 𝐾 into an algebraic closure of 𝐹 (so over a set
of coset representatives for 𝐻 in Gal(𝐿/𝐹 ) by the Fundamental Theorem of Galois Theory). This is a
product of Galois conjugates of 𝛼. In particular, if 𝐾 /𝐹 is Galois this is ∏𝜎∈Gal(𝐾 /𝐹 ) 𝜎(𝛼)
Solution.
(a) Let Ω = {𝜎 ∣ 𝜎𝐻 = 𝐻 , 𝐻 ≤ Gal(𝐿/𝐹 )} then 𝑁𝐾 /𝐹 (𝛼) = ∏𝜎∈Ω 𝜎(𝛼). Showing that 𝑁𝐾 /𝐹 (𝛼) ∈ 𝐹 is analogous
to showing that any 𝜏 ∈ Gal(𝐿/𝐹 ) fixes 𝑁𝐾 /𝐹 (𝛼) as 𝐹 is the fixed field of Gal(𝐿/𝐹 ).
Now, let 𝜏 ∈ Gal(𝐿/𝐹 ) we then have
We can now notice that if 𝜏𝜎1 is in the same coset as 𝜏𝜎2 then 𝜏𝜎1 = 𝜏𝜎1 ℎ, ℎ ∈ 𝐻 which implies that 𝜎1 is
in the same coset as 𝜎2 , therefore {𝜎 ∣ 𝜎𝐻 = 𝐻 } = {𝜏𝜎 ∣ 𝜏𝜎𝐻 = 𝐻 } = Ω. Hence we can simplify
√
(c) If 𝐾 = 𝐹 ( 𝐷) is a quadratic
√ extension
√ then we have 2 embeddings. Namely, 𝜎, 𝜏 where 𝜎 is identity and
𝜏 which fixes 𝐹 and maps 𝐷 ↦ − 𝐷, hence
√ √ √ √ √
𝑁𝐾 /𝐹 (𝑎 + 𝐷) = 𝜎(𝑎 + 𝑏 𝐷)𝜏(𝑎 + 𝑏 𝐷) = (𝑎 + 𝑏 𝐷)(𝑎 − 𝑏 𝐷) = 𝑎2 − 𝑏2 𝐷
(d) 𝑚𝛼 (𝑥) has degree 𝑑 and therefore [𝐹 (𝛼) ∶ 𝐹 ] = 𝑑 which divides [𝐾 ∶ 𝐹 ] = 𝑛. Let 𝑚𝛼 (𝑥) = 𝑥 𝑑 + 𝑎𝑑−1 𝑥 𝑑−1 +
⋯ + 𝑎0 , where 𝑎𝑖 ∈ 𝐹 . Consider 𝐻 = {𝜎 ∈ 𝐺 ∣ 𝜎(𝛼) = 𝛼} and notice that it is a subgroup of 𝐺. For any
𝜎 ∈ 𝐺, it must be that 𝜎 ∶ 𝛼 ↦ 𝛼𝑖 , where 𝛼1 = 𝛼, … , 𝛼𝑑 are the roots of 𝑚𝛼 (𝑥). Since 𝐾 /𝐹 is Galois, then
any irreducible polynomial over 𝐹 is separable, and thus we can conclude that the 𝛼𝑖 ’s are distinct.
Now consider 𝐺 acting on 𝐾 in the obvious way (That is 𝜎 ⋅ 𝛼 = 𝜎(𝛼)). Then notice that 𝐻 = Stab(𝛼), and
by orbit-stabiliser theorem, we have
|𝐺| = |𝐻 ||(𝛼)|
𝑛 = |𝐻 |(𝑑) there are 𝑑 distinct roots
𝑛
|𝐻 | =
𝑑
10
Then
𝑁𝐾 /𝐹 (𝛼) = ∏ 𝜎(𝛼)
𝜎∈𝐺
𝑑
= ∏ ∏(𝜎𝑖 𝜏)(𝛼)
𝑖=1 𝜏∈𝐻
𝑑
𝑛
= ∏ (𝜎𝑖 (𝛼)) 𝑑 𝜏(𝛼) = 𝛼, ∀𝜏 ∈ 𝐻
𝑖=1
𝑑 𝑛
= ∏ 𝛼𝑖𝑑
𝑖=1
𝑛 𝑛 𝑛
Since 𝑎0 = (−1)𝑑 ∏𝑑𝑖=1 𝛼𝑖 , then it follows that 𝑁𝐾 /𝐹 (𝛼) = (∏𝑑𝑖=1 𝛼𝑖 ) 𝑑 = ((−1)𝑑 𝑎0 ) 𝑑 = (−1)𝑛 𝑎0𝑑 as desired.
□
3 Chapter 14.3
Question 3.1
Question 3.2
Solution. We know 𝑥 4 − 𝑥 = 𝑥(𝑥 − 1)(𝑥 2 + 𝑥 + 1) and 𝑔(𝑥) = 𝑥 2 + 𝑥 + 1 is irreducible in 𝔽2 [𝑥]. Let 𝜃 be a root
of 𝑔(𝑥), we then have
𝔽4 ≅ 𝔽2 [𝑥]/(𝑥 2 + 𝑥 + 1) ≅ 𝔽2 (𝜃) = {𝑎 + 𝑏𝜃 ∣ 𝑎, 𝑏 ∈ 𝔽2 } = {0, 1, 𝜃, 1 + 𝜃}
Using 𝜃2 + 𝜃 + 1 = 0 we then have the multiplication table:
× 0 1 𝜃 𝜃+1
0 0 0 0 0
1 0 1 𝜃 𝜃+1
𝜃 0 𝜃 𝜃+1 1
𝜃+1 0 𝜃+1 1 𝜃
From Question 3.1 we have 𝑥 8 − 𝑥 = 𝑥(𝑥 − 1)(𝑥 3 + 𝑥 + 1)(𝑥 3 + 𝑥 2 + 1) and ℎ(𝑥) = 𝑥 3 + 𝑥 + 1 is irreducible in
𝔽2 [𝑥], let 𝛼 be a root of ℎ(𝑥). We then have
𝔽8 ≅ 𝔽2 (𝛼) ≅ 𝔽2 [𝑥]/(𝑥 3 + 𝑥 + 1) ≅ {𝑎 + 𝑏𝛼 + 𝑐𝛼 2 ∣ 𝑎, 𝑏, 𝑐 ∈ 𝔽2 } = {0, 1, 𝛼, 𝛼 + 1, 𝛼 2 , 𝛼 2 + 1, 𝛼 2 + 𝛼, 𝛼 2 + 𝛼 + 1}
Using 𝛼 3 + 𝛼 + 1 = 0, we have the multiplication table:
× 0 1 𝛼 𝛼+1 𝛼2 𝛼2 + 1 𝛼2 + 𝛼 𝛼2 + 𝛼 + 1
0 0 0 0 0 0 0 0 0
1 0 1 𝛼 𝛼+1 𝛼2 𝛼2 + 1 𝛼2 + 𝛼 𝛼2 + 𝛼 + 1
𝛼 0 𝛼 𝛼2 𝛼2 + 𝛼 𝛼+1 1 𝛼2 + 𝛼 + 1 𝛼2 + 1
2
𝛼+1 0 𝛼+1 𝛼 +𝛼 𝛼2 + 1 𝛼2 + 𝛼 + 1 𝛼2 1 𝛼
𝛼2 0 𝛼2 𝛼+1 2
𝛼 +𝛼+1 𝛼2 + 𝛼 𝛼 𝛼2 + 1 1
2 2
𝛼 +1 0 𝛼 +1 1 𝛼2 𝛼 𝛼2 + 𝛼 + 1 𝛼+1 𝛼2 + 𝛼
𝛼2 + 𝛼 0 𝛼2 + 𝛼 𝛼2 + 𝛼 + 1 1 𝛼2 + 1 𝛼+1 𝛼 𝛼2
𝛼2 + 𝛼 + 1 0 𝛼2 + 𝛼 + 1 𝛼2 + 1 𝛼 1 𝛼2 + 𝛼 𝛼2 𝛼+1
11
□
Question 3.3
Solution.
𝑛
(Method 1) Suppose 𝐾 is a finite algebraically closed field, then 𝐾 ≅ 𝔽𝑝𝑛 = {𝛼 ∣ 𝛼 𝑝 − 𝛼 = 0}. Let 𝛼0 , 𝛼1 ⋯ 𝛼𝑛
be the distinct roots and hence all the elements of 𝐾 , then 𝑓 (𝑥) = 1 + ∏𝑛𝑖=0 (𝑥 − 𝛼𝑖 ) has no root in
𝐾 [𝑥] which contradicts the assumption that 𝐾 is algebraically closed.
𝑚
(Method 2) Alternatively, for a field to be algebraically closed, it necessarily must contain roots of 𝑥 𝑝 − 𝑥 for
𝑚
any 𝑚 and for any prime 𝑝. Since each 𝑥 𝑝 − 𝑥 has 𝑝𝑚 distinct roots, then |𝔽| ≥ 𝑝𝑚 for any 𝑝, 𝑚.
That is, it must be infinite.
(Method 3) Alternatively, we proceed by contraposition. Fix some arbitrary finite field 𝔽𝑝𝑛 . Let 𝑞 be a prime
s.t. 𝑞 ∤ 𝑛. By proposition 17, ∃𝑄(𝑥) ∈ 𝔽𝑝 irreducible and of degree 𝑞. Fix any 𝛼 ∈ 𝔽𝑝𝑛 . If 𝑄(𝛼) = 0,
then we have the following.
𝔽𝑝 ⊆ 𝔽𝑝 (𝛼) ⊆ 𝔽𝑝𝑛
where the degree of the first extension is 𝑞. But 𝑞 ∤ 𝑛 and thus cannot be the case.
Question 3.4
Construct the finite field of 16 elements and find a generator for the multiplicative group. How many
generators are there?
Solution. A finite field with 16 elements will be isomorphic to 𝔽24 . Again by the discussion of Proposition 18
we have 𝑥 16 − 𝑥 = 𝑥(𝑥 − 1)(𝑥 2 + 𝑥 + 1)(𝑥 4 + 𝑥 3 + 1)(𝑥 4 + 𝑥 + 1)(𝑥 4 + 𝑥 3 + 𝑥 2 + 𝑥 + 1) and 𝑓 (𝑥) = 𝑥 4 + 𝑥 + 1 is
irreducible in 𝔽2 [𝑥], let 𝜃 be a root of 𝑓 (𝑥), hence we have
Now we can notice that 𝑥 3 ≠ 𝑥, 𝑥 5 = 𝑥 + 𝑥 2 ≠ 𝑥 hence ord(𝑥) ≠ 3 or 5 but it must divide 15 = |𝔽×16 |,
hence ord(𝑥) = 15 and ⟨𝑥⟩ generates 𝔽×16 , therefore we conclude ⟨𝑥⟩ ≅ ℤ15 and hence there will be 𝜑(15) = 8
generators, they are {𝑥 𝑎 ∣ gcd(𝑎, 15) = 1} = {𝑥 1 , 𝑥 2 , 𝑥 4 , 𝑥 7 , 𝑥 8 , 𝑥 11 , 𝑥 13 , 𝑥 14 } □
Question 3.5
Solution. Notice that 𝑓 (𝑥) = 𝑥 3 − 𝑥 + 1 and 𝑔(𝑥) = 𝑥 3 − 𝑥 − 1 are both irreducible in 𝔽3 [𝑥] because
𝑓 (0) = 𝑓 (1) = 𝑓 (2) = 1 ≠ 0 and 𝑔(0) = 𝑔(1) = 𝑔(2) = −1 = 2 ≠ 0, therefore we have
Let 𝛼(𝑥) = 𝑎𝑥 2 + 𝑏𝑥 + 𝑐 be a root of 𝑓 (𝑥) in 𝔽3 [𝑥]/(𝑔(𝑥)), then if we map 𝑥 ∈ 𝔽3 /(𝑓 (𝑥)) ↦ 𝛼(𝑥) we have our
12
isomorphism. Now, we need to find 𝛼(𝑥) such that 𝑓 (𝛼(𝑥)) = 0
Therefore we have 𝑎 = 0, 𝑏 = 2, then we just let 𝑐 = 0 and we have 𝛼(𝑥) = 2𝑥, we finally have the explicit
isomorphism
√ √ √
Suppose
√ 𝐾√ = ℚ(𝜃) = ℚ( 𝐷1 , 𝐷2 ) with 𝐷1 , 𝐷2 ∈ ℤ, is a biquadratic extension and that 𝜃 = 𝑎 + 𝑏 𝐷1 +
𝑐 𝐷2 + 𝑑 𝐷1 𝐷2 where 𝑎, 𝑏, 𝑐, 𝑑 ∈ ℤ are integers. Prove that the minimal polynomial 𝑚𝜃 (𝑥) for 𝜃 over
ℚ is irreducible of degree 4 over ℚ but is reducible modulo every prime 𝑝. In particular show that the
polynomial 𝑥 4 − 10𝑥 2 + 1 is irreducible in ℤ[𝑥] but is reducible modulo every prime. [Use the fact that
there are no biquadratic extensions over finite fields.]
Question 3.7
Prove that one of 2, 3 or 6 is a square in 𝔽𝑝 for every prime 𝑝. Conclude that the polynomial
Solution. Let ⟨𝑥⟩ = 𝔽×𝑝 . Then 𝑎 ∈ 𝔽×𝑝 is a square if and only if 𝑎 = 𝑥 2𝑘 = (𝑥 𝑘 )2 where 𝑏 ∈ 𝔽×𝑝 , 𝑘 ∈ ℤ. Now
suppose 2 and 3 are not square in 𝔽𝑝 , this means 2 = 𝑥 2𝑘1 +1 and 3 = 𝑥 2𝑘2 +1 where 𝑘1 , 𝑘2 ∈ ℤ and therefore
6 = 𝑥 2(𝑘1 +𝑘2 +1) is a square in 𝔽𝑝 .
Therefore, 2, 3 or 6 must be a square in 𝔽𝑝 and hence WLOG suppose 2 is a square in 𝔽𝑝 there is an 𝑎 ∈ 𝔽𝑝
such that 𝑎2 − 2 =√0 which √ implies
√ 𝑎 is a root of (𝑥 2 − 2)(𝑥 2 − 3)(𝑥 2 − 6). Furthermore, the real roots of this
polynomial are ± 2, ± 3, ± 6 which are all not integers, hence the polynomial has no root in ℤ. □
13